LSAT and Law School Admissions Forum

Get expert LSAT preparation and law school admissions advice from PowerScore Test Preparation.

 z.em
  • Posts: 10
  • Joined: Aug 20, 2016
|
#29515
Hello,

First, I wanted to know why M's statement implies that left handed people do not live as long as right handed people. How do you make that inference?--it could just be a coincidence that one cannot find left-handed 85-90 year olds and are able to find right handed people. Second, how is answer choice D weakened by Q's response? Is it because it says BOTH genetic predisposition and societal pressures, because Q seems to imply that it is due to societal pressure--the fear of punishment?

Thank you.
 David Boyle
PowerScore Staff
  • PowerScore Staff
  • Posts: 836
  • Joined: Jun 07, 2013
|
#29529
z.em wrote:Hello,

First, I wanted to know why M's statement implies that left handed people do not live as long as right handed people. How do you make that inference?--it could just be a coincidence that one cannot find left-handed 85-90 year olds and are able to find right handed people. Second, how is answer choice D weakened by Q's response? Is it because it says BOTH genetic predisposition and societal pressures, because Q seems to imply that it is due to societal pressure--the fear of punishment?

Thank you.

Hello z.em,

There may not be a direct implication or strong inference, although since we know that there are many people between 85-90, then if few are left-handed, that may tend to imply that most of the left-handers died by that point.
And, you seem to be on the right track, that Q more emphasizes the social issue, not the genetic issue.

Hope this helps,
David
 PositiveThinker
  • Posts: 49
  • Joined: Dec 24, 2016
|
#34766
Administrator wrote:Complete Question Explanation

Cannot Be True. The correct answer choice is (A)

This stimulus provides a dialogue between M and Q. M points out that there are not many left-handed people who have survived to 85-90 years of age (the implication being that something causes the left-handed to die sooner). Q responds with an alternative explanation: back when 85 year olds were children, left-handedness was discouraged.

The challenging question stem basically asks what Q is arguing against—it is M’s possibly implied assertion that there is something causing the left-handed to die sooner.

Only correct answer choice (A) reflects the argument that Q is pro-actively disputing with an alternative cause. Q is arguing against the implication that the left-handed have some sort of disadvantage causing them not to survive as long.



I just don't see how you made the jump to assume that M implied that all the left handed 85 to 90 year olds were all dead. Very very few questions have frustrated me like this one. And this question stem language should be PROHIBITED. This test is already hard enough without me having to learn hieroglyphics just to parse out a question.
 Kristina Moen
PowerScore Staff
  • PowerScore Staff
  • Posts: 230
  • Joined: Nov 17, 2016
|
#34934
Hi PositiveThinker,

You're right, the language in this question stem is a bit confusing. Keep in mind that this from a 1992 test - it was administered 25 years ago! Remember when you review old tests that sometimes the question stems might look a bit different from what you see on current tests (and what you'll see on your test!). That should be a relief, I hope.

But it useful to look at this question stem, as it gets your brain thinking "out of the box." If you've been studying, most of the question stems you encounter will be familiar to you. This question stem doesn't fit neatly into one of our question types. So let's discuss what we're being asked:

Stem (emphasis added): Q's response [Children were punished for being left-handed] serves to counter any use by M of the evidence about 85 to 90 year olds [It is almost impossible to find a person between the ages of 85 and 90 who primarily uses the left hand] in support of which one of the following hypotheses?

Basically, what could M NO LONGER say now that Q has given his evidence? What assertion does M's evidence no longer back up? While this doesn't fit neatly into a question type, you could think of this as a Cannot Be True. Now that we have Q's evidence that children used to be punished for being left-handed, what assertion is no longer valid? Look at answer choices (B) through (E). All of those assertions could be supported by both Q's and M's evidence. But look at answer choice (A) again. While that assertion is supported by M's evidence, it is NOT supported by Q's evidence.

M does not imply all left-handed people over 85 are dead. But his evidence (that's it almost impossible to find left-handed folks in that age range) does support the assertion that "Being born right-handed confers a survival advantage." Q responds with a reason why that might be, and it has nothing to do with survival advantage, but social reasons. So M can no longer claim that "being born right-handed confers a survival advantage." It's not that M made that claim in the stimulus, but the question stem is about POSSIBLE claims/assertions that M might make.

Hope this helps. And if this question still twists you in knots, move on. You are unlikely to see a question stem worded like this in the future.

Get the most out of your LSAT Prep Plus subscription.

Analyze and track your performance with our Testing and Analytics Package.